0 Daumen
1,5k Aufrufe
Berechnen Sie den Grenzwert bis auf einen Fehler vom Betrag \(<10^{-4}\). Wie viele Reihenglieder benötigen Sie hierfür?

Gegeben ist \(\sum_{k=0}^{\infty}{\frac{(3i)^k}{k!}}\). 

Wie gehe ich hier vor? 

Mein Versuch

Im Skript steht ein Satz:

Sei \((a_k)_k\) eine monoton fallende Nullfolge in \(\mathbb{R}\). Dann ist für \(z\in \mathbb{C}\) mit \(|z|\leq 1\) und \(z\neq 1\) die Reihe \(\sum_{k=1}^{\infty}{a_kz^k}\) konvergent und es gilt die Restgliedabschätzung:$$\left\lvert \sum_{k=n}^{\infty}{a_kz^k}\right\rvert\leq \frac{2a_n}{|z-1|}$$ für \(n\in \mathbb{N}\).

Das Problem ist, dass \(|3i|=3>1\) und man den Satz deshalb nicht anwenden kann. Ich war bisher so weit, dass man dem Satz aber etwas wichtiges entnehmen kann und das auf den vorliegenden Fall anwenden kann:

\(a_k=\frac 1{k!}\) und \(k\geq n\):$$|a_{k+1} (3i)^{k+1}| = \frac{3}{k+1} |a_{k} (3i)^k| \le \frac{3}{n+1} |a_{k} (3i)^k|$$$$|a_k (3i)^k| \le \left(\frac3{n+1}\right)^{k-n} |a_n (3i)^n|$$$$\sum_{k=n}^\infty |a_k (3i)^k| \le \sum_{k=n}^\infty \left(\frac3{n+1}\right)^{k-n} |a_n (3i)^n|$$ Für \(n\geq 3\) bedeutet das, dass $$\left| \sum_{k=n}^\infty a_k (3i)^k \right| \le\frac{1}{1-\frac{3}{n+1}} \frac{3^n}{n!}$$ Man braucht jetzt also ein \(n\geq 3\) so, dass: $$\frac{1}{1-\frac{3}{n+1}} \frac{3^n}{n!} \le 10^{-4}$$

⇒ Man braucht ca. 14 Reihenglieder!

Bemerkung:

Das geht aber nur mit WolframAlpha, ansonsten kann man die Ungleichung nicht lösen! (schlecht!)

Avatar von 28 k

Hey racine_carrée,

ich habe leider keine wirkliche Lösung auf deine Frage, aber hast du zufällig die Nummer 1 auf dem Übungsblatt?

bitte nicht als "Antwort" auf diese Frage. Besser: als Kommentar. Ich habe das nun verschoben. Ja, ich konnte Aufgabe 1) lösen.

@EmNero

möchtest Du Deinen Kommentar vielleicht als Antwort posten?

a) kann ich dann einen Stern verteilen

b) ist die Frage dann geschlossen.

Wenn dir der Kommentar als Antwort genügt darf ihn ein Admin gerne in eine Antwort umwandeln. Selbst kann ich das leider nicht machen.

Du kannst deinen Kommentar doch bearbeiten, oder nicht? Dann kopiere einfach alles raus und stelle es als Antwort rein.

2 Antworten

+2 Daumen
 
Beste Antwort

Ich auch nicht nicht so ganz :-) Lass mich das vielleicht mal ein bisschen sortieren:

Wir können den Satz anwenden für

Damit meine ich den Satz, den du oben gepostet hast.

Die Folge ist ja sicher eine monoton fallende Nullfolge (Index beginnt im Satz bei k=1) und der Betrag von i ist 1 und somit auch kleiner gleich 1

Also gilt nach deinem Satz für alle natürlichen Zahlen \( n \ge 1 \):

$$ \left\lvert\sum_{k=n}^\infty \frac{3^{k+2}}{(k+2)!} i^{k}\right\rvert = \left\lvert\sum_{k=n}^\infty \frac{3^{k+2}}{(k+2)!} i^{k+2}\right\rvert \le \frac{a_n}{\sqrt{2}} $$

Index shiften:

$$ \left\lvert\sum_{k=n+2}^\infty \frac{3^{k}}{k!} i^{k}\right\rvert \le \frac{a_{n}}{\sqrt{2}} $$

Bzw. für alle \( N \ge 3\):

$$ \left\lvert\sum_{k=N}^\infty \frac{3^{k}}{k!} i^{k}\right\rvert \le \frac{a_{N-2}}{\sqrt{2}} = \frac{3^{N-2+2}}{\sqrt{2}\cdot(N-2+2)!} = \frac{3^N}{\sqrt{2}\cdot N!} $$

---

Und jetzt weißt du, dass für \( N \ge 3 \) dein Fehler kleiner als

$$ \frac{3^N}{\sqrt{2}\cdot N!} $$

ist. Wenn man N=1 und N=2 ausschließen kann, dann kann man damit ein N suchen, s.d.

$$ \frac{3^N}{\sqrt{2}\cdot N!} \le 10^{-4}$$

gilt. Das ist eine etwas netter und bessere Abschätzung als die Ungleichung die du (und ich in meinem ersten Kommentar) durch Abschätzen mit der geometrischen Reihe erhalten hast:

$$ \frac{3^N}{\sqrt{2}\cdot N!} \quad\textrm{vs.}\quad \frac{1}{1-\frac{3}{N+1}}\frac{3^N}{N!} $$

Avatar von 6,0 k

Danke, das sieht schon netter aus! Das Ergebnis bleibt auch gleich. Es bleibt trotzdem unrealistisch diese Ungleichung im Kopf zu lösen...

@rc: Ich habe nun die beiden längeren Kommentare zu Antworten gemacht. Du als Moderator (Emnero ist das leider wahrscheinlich noch nicht) kannst, den Teil, der besser Kommentar sein sollte, in einen Kommentar umwandeln. Aufpassen: Option "zur Frage" bzw. "zur Antwort" von Anfang an richtig wählen. Umwandlung zurück in eine Antwort bräuchte dann mW nochmals jemanden mit Redakteur-Rechten.

+1 Daumen

Was darfst du denn zum lösen alles verwenden? Der Grenzwert ist ja \( e^{3i}\).

Du könntest natürlich abschätzen:

$$ \left\lvert \sum_{k=0}^\infty \frac{(3i)^k}{k!} - \sum_{k=0}^N \frac{(3i)^k}{k!}\right\rvert \\= \left\lvert  \sum_{k=N+1}^\infty \frac{(3i)^k}{k!}\right\rvert \\\le \sum_{k=N+1}^\infty \frac{|3i|^k}{k!}\\=\sum_{k=N+1}^\infty \frac{3^k}{k!}\\=\frac{3^{N+1}}{(N+1)!}\left(1+\sum_{k=1}^\infty \frac{3^k}{(N+2)\dotsm(k+N+1)} \right)$$

Und

$$ (N+2)\dotsm(k+N+1) \ge (N+2)^k $$

Also

$$ \left\lvert \sum_{k=0}^\infty \frac{(3i)^k}{k!} - \sum_{k=0}^N \frac{(3i)^k}{k!}\right\rvert \le \frac{3^{N+1}}{(N+1)!} \sum_{k=0}^\infty \left(\frac{3}{N+2}\right)^k$$

Avatar von 6,0 k

[zur Antwort hinzugefügt]

Wir können den Satz anwenden für

$$ a_k =\frac{3^{k+2}}{(k+2)!},\quad z=i$$

also gilt für alle n:

$$\left\lvert\sum_{k=n}^\infty \frac{3^{k+2}}{(k+2)!} i^{k+2}\right\rvert \le \frac{a_n}{\sqrt{2}}$$

Indexshift:

$$\left\lvert\sum_{k=n+2}^\infty \frac{3^{k}}{k!} i^{k}\right\rvert \le \frac{a_{n}}{\sqrt{2}}$$

Ich blicke gerade nicht ganz, was ich tun muss?!

Ein anderes Problem?

Stell deine Frage

Willkommen bei der Mathelounge! Stell deine Frage einfach und kostenlos

x
Made by a lovely community